Những câu hỏi liên quan
l҉o҉n҉g҉ d҉z҉
Xem chi tiết
alibaba nguyễn
31 tháng 3 2021 lúc 13:56

Câu hỏi của Trần Lê Nguyên Mạnh - Toán lớp 9 - Học trực tuyến OLM

Bình luận (0)
 Khách vãng lai đã xóa
Trần Lê Nguyên Mạnh
Xem chi tiết
Kiệt Nguyễn
23 tháng 8 2020 lúc 21:14

Áp dụng bất đẳng thức Bunyakovsky ta được:          \(\left(ab+bc+ca+1\right)\left(\frac{a}{b}+\frac{b}{c}+\frac{c}{a}+1\right)\ge\left(a+b+c+1\right)^2\)\(\left(ab+bc+ca+1\right)\left(\frac{b}{a}+\frac{c}{b}+\frac{a}{c}+1\right)\ge\left(b+c+a+1\right)^2\)

Cộng theo vế hai bất đẳng thức này ta được \(\left(ab+bc+ca+1\right)\frac{\left(a+b\right)\left(b+c\right)\left(c+a\right)}{abc}\ge2\left(a+b+c+1\right)^2\)hay \(\frac{ab+bc+ca+1}{\left(a+b+c+1\right)^2}\ge\frac{2abc}{\left(a+b\right)\left(b+c\right)\left(c+a\right)}\)

Đến đây, ta quy bất đẳng thức cần chứng minh về dạng:\(\frac{2abc}{\left(a+b\right)\left(b+c\right)\left(c+a\right)}+\frac{3}{8}\sqrt[3]{\frac{\left(a+b\right)\left(b+c\right)\left(c+a\right)}{abc}}\ge1\)

Áp dụng bất đẳng thức Cauchy ta được \(\frac{2abc}{\left(a+b\right)\left(b+c\right)\left(c+a\right)}+\frac{1}{8}\sqrt[3]{\frac{\left(a+b\right)\left(b+c\right)\left(c+a\right)}{abc}}\)\(\ge2\sqrt{\frac{2abc}{\left(a+b\right)\left(b+c\right)\left(c+a\right)}.\frac{1}{8}\sqrt[3]{\frac{\left(a+b\right)\left(b+c\right)\left(c+a\right)}{abc}}}\)\(=\sqrt{\sqrt[3]{\frac{a^2b^2c^2}{\left(a+b\right)^2\left(b+c\right)^2\left(c+a\right)^2}}}=\sqrt[3]{\frac{abc}{\left(a+b\right)\left(b+c\right)\left(c+a\right)}}\)(*)

Cũng theo bất đẳng thức Cauchy ta được \(\sqrt[3]{\frac{abc}{\left(a+b\right)\left(b+c\right)\left(c+a\right)}}+\frac{1}{4}\sqrt[3]{\frac{\left(a+b\right)\left(b+c\right)\left(c+a\right)}{abc}}\ge2\sqrt{\frac{1}{4}}=1\)(**)

Từ (*) và (**) suy ra được \(\frac{2abc}{\left(a+b\right)\left(b+c\right)\left(c+a\right)}+\frac{3}{8}\sqrt[3]{\frac{\left(a+b\right)\left(b+c\right)\left(c+a\right)}{abc}}\ge1\)

Vậy bất đẳng thức được chứng minh

Đẳng thức xảy ra a = b = c = 1

Bình luận (0)
 Khách vãng lai đã xóa
Tiến Nguyễn Minh
Xem chi tiết
Vũ Tiến Manh
21 tháng 10 2019 lúc 22:19

1) Áp dụng bunhiacopxki ta được \(\sqrt{\left(2a^2+b^2\right)\left(2a^2+c^2\right)}\ge\sqrt{\left(2a^2+bc\right)^2}=2a^2+bc\), tương tự với các mẫu ta được vế trái \(\le\frac{a^2}{2a^2+bc}+\frac{b^2}{2b^2+ac}+\frac{c^2}{2c^2+ab}\le1< =>\)\(1-\frac{bc}{2a^2+bc}+1-\frac{ac}{2b^2+ac}+1-\frac{ab}{2c^2+ab}\le2< =>\)

\(\frac{bc}{2a^2+bc}+\frac{ac}{2b^2+ac}+\frac{ab}{2c^2+ab}\ge1\)<=> \(\frac{b^2c^2}{2a^2bc+b^2c^2}+\frac{a^2c^2}{2b^2ac+a^2c^2}+\frac{a^2b^2}{2c^2ab+a^2b^2}\ge1\)  (1) 

áp dụng (x2 +y2 +z2)(m2+n2+p2\(\ge\left(xm+yn+zp\right)^2\)

(2a2bc +b2c2 + 2b2ac+a2c2 + 2c2ab+a2b2). VT\(\ge\left(bc+ca+ab\right)^2\)   <=> (ab+bc+ca)2. VT \(\ge\left(ab+bc+ca\right)^2< =>VT\ge1\)  ( vậy (1) đúng)

dấu '=' khi a=b=c

Bình luận (0)
 Khách vãng lai đã xóa
HD Film
21 tháng 10 2019 lúc 22:26

4b, \(\frac{a^3}{a^2+b^2}+\frac{b^3}{b^2+c^2}+\frac{c^3}{c^2+a^2}=1-\frac{ab^2}{a^2+b^2}+1-\frac{bc^2}{b^2+c^2}+1-\frac{ca^2}{a^2+c^2}\)

\(\ge3-\frac{ab^2}{2ab}-\frac{bc^2}{2bc}-\frac{ca^2}{2ac}=3-\frac{\left(a+b+c\right)}{2}=\frac{3}{2}\)

Bình luận (0)
 Khách vãng lai đã xóa
HD Film
21 tháng 10 2019 lúc 22:35

4c, 

\(\frac{a+1}{b^2+1}+\frac{b+1}{c^2+1}+\frac{c+1}{a^2+1}=a+b+c-\frac{b^2}{b^2+1}-\frac{c^2}{c^2+1}-\frac{a^2}{a^2+1}+3--\frac{b^2}{b^2+1}-\frac{c^2}{c^2+1}-\frac{a^2}{a^2+1}\)\(\ge6-2\cdot\frac{\left(a+b+c\right)}{2}=3\)

Bình luận (0)
 Khách vãng lai đã xóa
Kurosaki Akatsu
Xem chi tiết
Thắng Nguyễn
25 tháng 6 2017 lúc 21:20

ko cả biết BĐT AM-GM với C-S là gì còn hỏi bài này rảnh háng

Bình luận (0)
alibaba nguyễn
26 tháng 6 2017 lúc 9:25

Đề sai rồi. Nếu như là a, b, c dương thì giá trị nhỏ nhất của nó phải là 9 mới đúng. Còn để có GTNN như trên thì điều kiện là a, b, c không âm nhé. Mà bỏ đi e thi cái gì mà phải giải câu cỡ này. Cậu này mạnh lắm đấy không phải dạng thường đâu.

Bình luận (0)
Bùi Hữu Vinh
Xem chi tiết
Đặng Ngọc Quỳnh
26 tháng 2 2021 lúc 6:00

Theo bđt Cauchy - Schwart ta có:

\(\text{Σ}cyc\frac{c}{a^2\left(bc+1\right)}=\text{Σ}cyc\frac{\frac{1}{a^2}}{b+\frac{1}{c}}\ge\frac{\left(\frac{1}{a}+\frac{1}{b}+\frac{1}{c}\right)^2}{\frac{1}{a}+\frac{1}{b}+\frac{1}{c}+a+b+c}\)\(=\frac{\left(\frac{1}{a}+\frac{1}{b}+\frac{1}{c}\right)^2}{\frac{1}{a}+\frac{1}{b}+\frac{1}{c}+3}\)

\(=\frac{\left(ab+bc+ca\right)^2}{abc\left(ab+bc+ca\right)+3a^2b^2c^2}\)

Đặt \(ab+bc+ca=x;abc=y\).

Ta có: \(\frac{x^2}{xy+3y^2}\ge\frac{9}{x\left(1+y\right)}\Leftrightarrow x^3+x^3y\ge9xy+27y^2\)

\(\Leftrightarrow x\left(x^2-9y\right)+y\left(x^3-27y\right)\ge0\) ( luôn đúng )

Vậy BĐT đc CM. Dấu '=' xảy ra <=> a=b=c=1

Bình luận (0)
 Khách vãng lai đã xóa
Bùi Hữu Vinh
26 tháng 2 2021 lúc 22:54

sai rồi nhé bạn 

Bình luận (0)
 Khách vãng lai đã xóa
Bùi Hữu Vinh
26 tháng 2 2021 lúc 23:05

làm sao mà \(x\left(x^2-9y\right)+y\left(x^3-27y\right)\ge0\)lại luôn đúng

Bình luận (0)
 Khách vãng lai đã xóa
Rhider
Xem chi tiết
Rhider
19 tháng 12 2021 lúc 20:14

ai giỏi ạ

Bình luận (0)
l҉o҉n҉g҉ d҉z҉
Xem chi tiết
Inequalities
28 tháng 12 2020 lúc 20:32

Đề sai. Nếu chỗ căn vế phải mà là căn bậc 3 thì t sol cho

Bình luận (0)
 Khách vãng lai đã xóa
Thắng Nguyên
Xem chi tiết
Kiệt Nguyễn
28 tháng 8 2020 lúc 9:36

Áp dụng giả thiết và một đánh giá quen thuộc, ta được: \(16\left(a+b+c\right)\ge\frac{1}{a}+\frac{1}{b}+\frac{1}{c}=\frac{ab+bc+ca}{abc}=\frac{\left(ab+bc+ca\right)^2}{abc\left(ab+bc+ca\right)}\ge\frac{3\left(a+b+c\right)}{ab+bc+ca}\)hay \(\frac{1}{6\left(ab+bc+ca\right)}\le\frac{8}{9}\)

Đến đây, ta cần chứng minh \(\frac{1}{\left(a+b+\sqrt{2\left(a+c\right)}\right)^3}+\frac{1}{\left(b+c+\sqrt{2\left(b+a\right)}\right)^3}+\frac{1}{\left(c+a+\sqrt{2\left(c+b\right)}\right)^3}\le\frac{1}{6\left(ab+bc+ca\right)}\)

 Áp dụng bất đẳng thức Cauchy cho ba số dương ta có \(a+b+\sqrt{2\left(a+c\right)}=a+b+\sqrt{\frac{a+c}{2}}+\sqrt{\frac{a+c}{2}}\ge3\sqrt[3]{\frac{\left(a+b\right)\left(a+c\right)}{2}}\)hay \(\left(a+b+\sqrt{2\left(a+c\right)}\right)^3\ge\frac{27\left(a+b\right)\left(a+c\right)}{2}\Leftrightarrow\frac{1}{\left(a+b+2\sqrt{a+c}\right)^3}\le\frac{2}{27\left(a+b\right)\left(a+c\right)}\)

Hoàn toàn tương tự ta có \(\frac{1}{\left(b+c+2\sqrt{b+a}\right)^3}\le\frac{2}{27\left(b+c\right)\left(b+a\right)}\)\(\frac{1}{\left(c+a+2\sqrt{c+b}\right)^3}\le\frac{2}{27\left(c+a\right)\left(c+b\right)}\)

Cộng theo vế các bất đẳng thức trên ta được \(\frac{1}{\left(a+b+\sqrt{2\left(a+c\right)}\right)^3}+\frac{1}{\left(b+c+\sqrt{2\left(b+a\right)}\right)^3}+\frac{1}{\left(c+a+\sqrt{2\left(c+b\right)}\right)^3}\le\frac{4\left(a+b+c\right)}{27\left(a+b\right)\left(b+c\right)\left(c+a\right)}\)Phép chứng minh sẽ hoàn tất nếu ta chỉ ra được \(\frac{4\left(a+b+c\right)}{27\left(a+b\right)\left(b+c\right)\left(c+a\right)}\le\frac{1}{6\left(ab+bc+ca\right)}\)\(\Leftrightarrow\left(a+b\right)\left(b+c\right)\left(c+a\right)\ge\frac{8}{9}\left(ab+bc+ca\right)\left(a+b+c\right)\)

Đây là một đánh giá đúng, thật vậy: đặt a + b + c = p; ab + bc + ca = q; abc = r thì bất đẳng thức trên trở thành \(pq-r\ge\frac{8}{9}pq\Leftrightarrow\frac{1}{9}pq\ge r\)*đúng vì \(a+b+c\ge3\sqrt[3]{abc}\)\(ab+bc+ca\ge3\sqrt[3]{\left(abc\right)^2}\))

Vậy bất đẳng thức được chứng minh

Đẳng thức xảy ra khi \(a=b=c=\frac{1}{4}\)

Bình luận (0)
 Khách vãng lai đã xóa
Nguyễn Anh Tuấn
Xem chi tiết
vu duc thanh
8 tháng 7 2016 lúc 22:36

bài 2 thì bạn áp dụng bdt cô si với lựa chọn điểm rơi  hoặc bdt holder  ( nó giống kiểu bunhia ngược ) . bai 1 thi ap dung cai nay \(\frac{1}{x}+\frac{1}{y}>=\frac{1}{x+y}\)  câu 1 khó hơn nhưng bạn biết lựa chọn điểm rơi với áp dụng bdt phụ kia là ok .

Bình luận (0)
Thắng Nguyễn
9 tháng 7 2016 lúc 8:37

Bài 1:Đặt VT=A

Dùng BĐT \(\left(x+y+z\right)\left(\frac{1}{x}+\frac{1}{y}+\frac{1}{z}\right)\ge9\Rightarrow\frac{1}{x+y+z}\le\frac{1}{9}\left(\frac{1}{x}+\frac{1}{y}+\frac{1}{z}\right)x,y,z>0\)

Áp dụng vào bài toán trên với x=a+c;y=b+a;z=2b ta có:

\(\frac{ab}{a+3b+2c}=\frac{ab}{\left(a+c\right)+\left(b+c\right)+2b}\le\frac{ab}{9}\left(\frac{1}{a+c}+\frac{1}{b+c}+\frac{1}{2b}\right)\)

Tương tự với 2 cái còn lại

\(A\le\frac{1}{9}\left(\frac{bc+ac}{a+b}+\frac{bc+ab}{a+c}+\frac{ab+ac}{b+c}\right)+\frac{1}{18}\left(a+b+c\right)\)

\(\Rightarrow A\le\frac{1}{9}\left(a+b+c\right)+\frac{1}{18}\left(a+b+c\right)=\frac{a+b+c}{6}\)

Đẳng thức xảy ra khi a=b=c 

Bài 2:

Biến đổi BPT \(4\left(\frac{a^3}{\left(1+b\right)\left(1+c\right)}+\frac{b^3}{\left(1+c\right)\left(1+a\right)}+\frac{c^3}{\left(1+a\right)\left(1+b\right)}\right)\ge3\)

\(\Rightarrow\frac{a^3}{\left(1+b\right)\left(1+c\right)}+\frac{b^3}{\left(1+c\right)\left(1+a\right)}+\frac{c^3}{\left(1+a\right)\left(1+b\right)}\ge\frac{3}{4}\)

Dự đoán điểm rơi xảy ra khi a=b=c=1

\(\frac{a^3}{\left(1+b\right)\left(1+c\right)}+\frac{1+b}{8}+\frac{1+c}{8}\ge\frac{3a}{4}\)

Tương tự suy ra

\(VT\ge\frac{2\left(a+b+c\right)-3}{4}\ge\frac{2\cdot3\sqrt{abc}-3}{4}=\frac{3}{4}\)

Bình luận (0)